Vous êtes sur la page 1sur 18

ERRATA

Page 18
Line -4 and Line -6: Change O(h3) and O(h4) to O(h2)

Page 22
Line 6: Change \eta = 1/3 to \eta = 21/2 and change \xi = 3 to
\xi = 11/2

Download the update for Page 22 in pdf format (176 K)


Download the update for Page 22 in postscript format (484K)

Page 82
Scalar Multiplication Box, Line 2: Change "... denoted by
&alphaA, is defined ..." To "... denoted by &alphaA (or equivalently
A&alpha), is defined ..."

Page 95
Line 2: Correct the spelling of "introduced"

Download the update for Page 95 in pdf format (32K)


Download the update for Page 95 in postscript format (596K)

Page 107
Line 12: Change n to k to make the expression read Ak =
...
AA A (k times)
Page 109
Lines 2-5, Line 7, and Line 14: For the matrices (but not the
vectors) change P to T

Page 121
Line 6: Make A and B boldface.

Page 129
Last line before the exercises: Insert a space after "Example
5.12.1"

Page 137
Line 9: Remove the extraneous period (the first period in the
sentence).

Page 139
Lines 5 and 8: In the subscripts of the displayed equations,
enclose n-r and m-r in parentheses [i.e., write (n-r) and (m-r) ].

Page 155
The second half of the proof should be rewritten with the roles of
T
R and R interchanged as follows.

Page 169
Lines -1 and -6: Change the font on R from \alpha_i\in\cal R to
\alpha_i\in\Re
Page 173
Line -11: f should be a function from Rn into Rm (rather than from
R into Rn)
m

Download the update for Page 173 in pdf format (48K)


Download the update for Page 173 in postscript format (752K)

Page 176
Line 10: Delete the transpose superscript on y to make the
expression look like this.

Line 16: Change the expression after the last implication to read
as follows.

Download the update for Page 176 in pdf format (52K)


Download the update for Page 176 in postscript format (792K)

Page 178
Lines 10 and 12: Add the phrase "(transposed)" to the end of
each line.

Page 204
Line -16: Change "... m independent nodal equations." to
"... m-1 independent nodal equations."

Page 239
The discussion of the projector in the second bullet does not
match Figure 4.7.2. Change the discussion in the second bullet to
reference Exercises 3.3.4 and 3.4.1.

Page 239
Change captions to 8 point font in Figures 4.7.1, 4.7.2, and 4.7.3

Download the update for Page 239 in pdf format (220K)


Download the update for Page 239 in postscript format (1.9M)

Page 273
Last line: Change = to less than or equal. Replace the last three
lines with the following.
Download the update for Page 273 in pdf format (40K)
Download the update for Page 273 in postscript format (1M)

Page 274
Line 8 below the shaded box: Change the word "integers" to
"real numbers"

Download the update for Page 274 in pdf format (88K)


Download the update for Page 274 in postscript format (596K)

Page 278
Line 9, Exercise 5.1.12b: y should be divided by the q-norm of y
rather than x.
Line 9 should read as follows.

Download the update for Page 278 in pdf format (48K)


Download the update for Page 278 in postscript format (628K)

Page 279
Line 3: The (1,2)-entry in matrix A should be -1 instead of 1.

Download the update for Page 279 in pdf format (44K)


Download the update for Page 279 in postscript format (756K)

Page 281
Line 6 in the shaded box: Change to to read:

Download the update for Page 281 in pdf format (117K)


Download the update for Page 281 in postscript format (1.1M)

Page 283
Equation (5.2.13): Change V*V=I to VV*=I

Page 285
Exercise 5.2.6 (e): Change V*V=I to VV*=I
Page 288
Lines 4 and 5 in the proof: Replace with the following.

Download the update for Page 288 in pdf format (52K)

Page 289
Line -4 (in the text): Delete the exponent on ||x||. i.e., change
||x||2 to ||x||

Page 291
Example 5.3.4: The equation in the middle of the page should
read as follows.

Download the update for Page 291 in pdf format (36K)


Download the update for Page 291 in postscript format (704K)

Page 302
Line 6: Change 4/n&pi to 4/(n&pi)

Page 320
The proof that isometries are unitary is incorrect. Change the
proof to read as follows.

Download the update for Page 320 in pdf format (40K)


Download the update for Page 320 in postscript format (656K)
Page 328
Line -3 inside of the shaded box: Add a subscript z to the
boldface P to make the line read:

Download the update for Page 328 in pdf format (84K)


Download the update for Page 328 in postscript format (1M)

Page 333
First line below shaded box: Change "...rotations matrices..." to
"...rotation matrices..."

Page 354
Line 15: The formulas for computing the paramaters for a Jacobi
rotation are not correct. Replace them with the ones shown below.

Page 357
Last line in shaded box: Make it read as follows.

Note. Throughout this section entries are indexed from 0 to n-1.


For example, the upper left-hand entry of Fn is considered to be
in the (0,0) position (rather than the (1,1) position), and the lower
right-hand entry is in the (n-1, n-1) position. When the context
makes it clear, the subscript n on Fn is omitted.
Download the update for Page 357 in pdf format (52K)
Download the update for Page 357 in postscript format (912K)

Page 364
Line 3: Change 128/2 = 64 to 512/2 = 256.

Insert 1/2 in front of the matrix in the equation at the bottom of


the page

Page 373
Line 7 inside of the shaded box: The shape subscript on vector
D should be 2 j x1 instead of j+1x1

Last line inside of the shaded box: Change

to

Download the update for Page 373 in pdf format (44K)


Download the update for Page 373 in postscript format (804K)

Page 379
Exercise 5.8.12, Line 5: Change cj-k mod(n) to c(j-k) mod(n)

Page 383
Last line in shaded box: Add the parenthetical phrase (empty
set) after greek letter phi

Download the update for Page 383 in pdf format (176K)


Download the update for Page 383 in postscript format (228K)

Page 395
Entire page: The properties and their proofs have been changed
to be more precise and more rigorous.

Download the update for Page 395 in pdf format (52K)


Download the update for Page 395 in postscript format (608K)

Page 398
Example 5.10.3: Line 1 should read as follows.
Download the update for Page 398 in pdf format (40K)
Download the update for Page 398 in postscript format (648K)

Page 389
Figure 5.9.2: Delete the greek subscript gamma from each greek
theta

Download the update for Page 389 in pdf format (104K)


Download the update for Page 389 in postscript format (1.3M)

Page 398
Example 5.10.3: Line 1 should read as follows.

Download the update for Page 398 in pdf format (40K)


Download the update for Page 398 in postscript format (648K)

Page 403
Line 1 below the first box: Change "... is a single vector ..." to "...
is a single nonzero vector ..."

Page 414
Line 12 (last line in box): Change "... A is an orthogonal matrix."
to "... A is a multiple of an orthogonal matrix."

Page 419
Line -9: Change qn to qm in the query vector qT

Page 429
First line below the shaded box: Remove an extra "illustrated"

Page 435
Last Line: Change the last term in this line from ||p-m||22 to
2
||b-p||2

Page 442
Problem 5.13.17 (b): There are missing brackets. The
expression should be: p=b-((u^Tb-beta)/uTu)u.
Problem 5.13.18 (b): The expression should be: p=b-
(u^Tb/u^Tw)w.

Problem 5.13.18 (c): The expression should be: p=b-((u^Tb-


beta)/u^Tw)w.

Page 459
Paragraph 2, Line 7: Change Liebniz to Leibniz.

Download the update for Page 459 in pdf format (84K)


Download the update for Page 459 in postscript format (196K)

Page 483
Exercise 6.2.6: Change: cnx1 to cmx1

Page 497
Line 9: Change: x to xT

Page 498
Line 2: Change: "... are contained the union ..." To: "... are
contained in the union ..."

Page 504
Line 1: Make the subscript on greek xi lower case. The line
should read

Download the update for Page 504 in pdf format (100K)


Download the update for Page 504 in postscript format (740K)

Page 510
Line 11: Delete extra space between p. and 590

Download the update for Page 510 in pdf format (116K)


Download the update for Page 510 in postscript format (1.1M)

Page 521
Exercise 7.2.7 is incorrectly worded. Replace it as shown below.
Page 527
Example 7.3.1, Line2: Change lower limit of summation in
Neumann series expansion from k=1 to k=0

Page 534
Footnote, last line: Change "(1910-)" to "(1910-2001)"
(Wielandt died on February 14, 2001)

Download the update for Page in pdf format (48K)


Download the update for Page in postscript format (716K)

Page 548
Line 5: Change = U = to =

Page 549
Line 6: Add the parenthetical remark (all quantities in the proof
on p. 547 become real when "normal" is replaced by "real-symmetric")
at the end of the first sentence of the second paragraph.

Page 562
Lines 13, 15, and 19: Change "fundamental mode(s)" to "normal
mode(s)"

Line 16: Reverse the signs in lambda_2 and lambda_3 to read

Download the update for Page 562 in pdf format (52K)


Download the update for Page 562 in postscript format (1M)

Page 570
Line 4: The Hessian term in the display should be divided by 2.
The correct formula is
Download the update for Page 570 in pdf format (56K)
Download the update for Page 570 in postscript format (1M)

Page 571
Lines 15 and 27: Change "fundamental mode(s)" to "normal
mode(s)"

Download the update for Page 571 in pdf format (48K)


Download the update for Page 571 in postscript format (824K)

Page 574
Lines 9 and 10: The last sentence of the first paragraph should
read
"... for every singular matrix A of index k with rank(Ak) = r,
there is ..."

Page 578
Lines 9, 10, and 11: Change X4 to X4

Page 598
Line 13, Exercise 7.8.11: Reverse the roles of the exponents m
and n
The correct statement is

Page 615
Change Exercise 7.9.22 as follows.
Page 627
Line -3: Change 7.10.28 to 7.10.25

Download the update for Page 627 in pdf format (100K)


Download the update for Page 627 in postscript format (784K)

Page 648
Remove the "det" in front of (xI - N)-1 in the equation on the
middle of the page

Page 659
Exercise 7.11.4: Change m(x) and c(x) as follows.
m(x) = (x - λ)(x - μ)2
c(x) = (x - λ)3(x - μ)3

Pages 663-686
A complete update to Sections 8.2 & 8.3

All aspects regarding the Collatz-Wielandt formula in Sections 8.2


and 8.3 have been changed (including exercises). In particular,
the proof of the Collatz-Wielandt formula is greatly simplified, and
incorrect statements are repaired.
Download the update for Sections 8.2 and 8.3 in pdf format (216K)
Download the update for Sections 8.2 and 8.3 in postscript format
(1.9M)
Note: In addition to other minor corrections, this update corrects the
following known errors.

Page 663
Statement (8.2.5) is not correct---remove it.

Page 664
Line 4: Make z boldface by changing Bz > z to Bz
> z.

Page 666
Change the proof of the Collatz-Wielandt formula to
read as follows.
Page 670
Line -6: Changing p to p*.

Page 674
Line 2 in Example 8.3.1: Add "z is nonzero" to the
supposition.
Line 7: Change the phrase "A ≥ 0 and x > 0 forces
Ax > 0" to read "A ≥ 0, A ≠ 0, and x > 0 forces some
(Ax)i > 0"

Page 678
Line 7 in Example 8.3.3: Replace 1 with 0. That is,
say "roots of unity sum to 0"

Page 683
Line -4: Change it's to its.

Page 691
Line 12: Change (8.3.8) to (8.3.10)
(Note: Make this change only if you installed the update for
Sections 8.2 & 8.3 on pages 663-686.)

Download the update for Page 691 in pdf format (40K)


Download the update for Page 691 in postscript format (660K)

Page 695
Line 5, equation (8.4.6): Change Prr to P1r in the first row of
matrix P in (8.4.6).

Page 697
Line 12, equation (8.4.8): Change Prr to P1r in the first row
of T11 in (8.4.8).

INDEX Page 708


Line 36: Add the entry --- empty set, 383

Download the update for Page 708 in pdf format (56K)


Download the update for Page 708 in postscript format (160K)

INDEX Page 709


Change "fundamental mode of vibration" to "fundamental
(normal) mode of vibration"

Download the update for Page 709 in pdf format (16K)


Download the update for Page 709 in postscript format (164K)

INDEX Page 713


Insert a new entry into the index -- "normal modes of vibration,
562, 571"

Download the update for Page 713 in pdf format (24K)


Download the update for Page 713 in postscript format (332K)

SOLUTIONS MANUAL Page 25


Solution 3.10.10 The Cholesky factor is upper triangular, and it is
the transpose of the matrix given.

SOLUTIONS MANUAL Page 42


Solution 4.7.4 The solution is missing. It will be included in the
next edition.

SOLUTIONS MANUAL Page 53


Solution 5.2.1 part c: The answer is 9 instead of sqrt{9}

SOLUTIONS MANUAL Page 54


Solution 5.2.6 part e: Change V*V=I to VV*=I

SOLUTIONS MANUAL Page 71


Solution 5.8.2 part b: The answer is e4 instead of 4e4

SOLUTIONS MANUAL Page 73


Solution 5.8.6 parts b and c: Reverse the order of the
components in the vectors on the left side of the equations in (b) and (c)-
--right side is okay.

SOLUTIONS MANUAL Page 92


Solution 5.10.12 parts (b) implies (c), (c) implies (d), and (d)
implies (e) should read as follows.
SOLUTIONS MANUAL Page 129
Solution 7.1.22 part b: Make all subscripts on greek xi lower
case. The expression should read

Download the update for Solutions Manual Page 129 in pdf format
(32K)
Download the update for Solutions Manual Page 129 in postscript format
(480K)

SOLUTIONS MANUAL Page 132


Solution 7.2.6: The constant term in the characteristic equation
should be 9/10 instead of 1/2.

SOLUTIONS MANUAL Page 132-133


Solution 7.2.7: Exercise 7.2.7 has been reworded---see Errata
for pg. 521. New solution is given below.
SOLUTIONS MANUAL Page 134
Solution 7.2.14: Last line of display: Change < alg multA to =
alg multA

SOLUTIONS MANUAL Page 139


Solution 7.3.9: Change the last five sentences to read as
follows.

SOLUTIONS MANUAL Page 145


Last Line: Reverse the signs of the entries in matrix K. The
correct matrix is

SOLUTIONS MANUAL Page 146


Line 1: Change "fundamental modes" to "normal modes"

Solution to Exercise 7.6.4: Change 18u2 + 8v2 = 72 to


8u + 18v2 = 72
2

SOLUTIONS MANUAL Page 149


Solution to Exercise 7.7.7, Line 6: Change dim R(L) = 2 = dim
R(L) to dim R(L) = 2 = dim N(L)

SOLUTIONS MANUAL Page 154


Solution to Exercise 7.9.5: Change f(z) = zn to f(z) = zk

Solution to Exercise 7.9.6: The (3,3)-entry in the matrix eA


should be 7e4 - e2 (the 7 was missing).

SOLUTIONS MANUAL Page 155


Solution to Exercise 7.9.9: The answer to the second part of the
question is "no." Consider a 2x2 matrix with eigenvalues 1 and -1 along
with the function f(x) = x2.
SOLUTIONS MANUAL Page 158
Solutions to Exercises 7.9.22 and 7.9.23 should be changed as
follows.

SOLUTIONS MANUAL Page 161


Solution to Exercise 7.10.9, Line 5: Change |1+w|n to |1-
n
w|

SOLUTIONS MANUAL Page 165


Solution to Exercise 7.11.4: The problem has been changed
(see above). The solution is changed by removing one μ from the
diagonal.

SOLUTIONS MANUAL Pages 167-171


Solutions to exercises in Chapter 8 have changed.
Problems have been added and deleted in the revision of
sections 8.2 and 8.3.

Download the update for Solutions Manual Pages 167-171 in pdf format
(76K)
Download the update for Solutions Manual Pages 167-171 in postscript
format (876K)

Back Cover
Correct the spelling of Mathematics in SIAM's address.

Please send other errors that you have discovered or suggest


improvements.

Vous aimerez peut-être aussi